How to choose $B$ to have $ operatorname{trace}(H) ge operatorname{trace}(B^{top}HB)$?












1












$begingroup$


Considering $H in mathbb{R}^{N times N}$ a real symmetric matrix which has both positive and negative eigenvalues, and $B in mathbb{R}^{N times M}$ a real matrix with positive entries.



Can I find a condition to make $ operatorname{trace}(H) ge operatorname{trace}(B^{top}HB)$?










share|cite|improve this question











$endgroup$








  • 1




    $begingroup$
    Hint : You can suppose that H is diagonal. How ?
    $endgroup$
    – DLeMeur
    Jan 20 at 19:11










  • $begingroup$
    @DLeMeur. Well, I fiddled around with your hint, but couldn't get the answer! :D
    $endgroup$
    – Babak
    Jan 20 at 20:53










  • $begingroup$
    Rehint: How is the trace of $H $ related to its diagonalization?
    $endgroup$
    – Javi
    Jan 21 at 13:20












  • $begingroup$
    @Javi: Yes, i know the trace of $H$ is the sum of the diagonal entries of the diagonalized matrix, but i cannot link it to the rhs. Does it mean that $B^top HB$ has the same eigenvalues as of $H$ for any given tall matrix $B$?
    $endgroup$
    – Babak
    Jan 22 at 11:07










  • $begingroup$
    No, but you could use that to obtain some sufficient conditions for your inequality to hold
    $endgroup$
    – Javi
    Jan 23 at 2:55
















1












$begingroup$


Considering $H in mathbb{R}^{N times N}$ a real symmetric matrix which has both positive and negative eigenvalues, and $B in mathbb{R}^{N times M}$ a real matrix with positive entries.



Can I find a condition to make $ operatorname{trace}(H) ge operatorname{trace}(B^{top}HB)$?










share|cite|improve this question











$endgroup$








  • 1




    $begingroup$
    Hint : You can suppose that H is diagonal. How ?
    $endgroup$
    – DLeMeur
    Jan 20 at 19:11










  • $begingroup$
    @DLeMeur. Well, I fiddled around with your hint, but couldn't get the answer! :D
    $endgroup$
    – Babak
    Jan 20 at 20:53










  • $begingroup$
    Rehint: How is the trace of $H $ related to its diagonalization?
    $endgroup$
    – Javi
    Jan 21 at 13:20












  • $begingroup$
    @Javi: Yes, i know the trace of $H$ is the sum of the diagonal entries of the diagonalized matrix, but i cannot link it to the rhs. Does it mean that $B^top HB$ has the same eigenvalues as of $H$ for any given tall matrix $B$?
    $endgroup$
    – Babak
    Jan 22 at 11:07










  • $begingroup$
    No, but you could use that to obtain some sufficient conditions for your inequality to hold
    $endgroup$
    – Javi
    Jan 23 at 2:55














1












1








1





$begingroup$


Considering $H in mathbb{R}^{N times N}$ a real symmetric matrix which has both positive and negative eigenvalues, and $B in mathbb{R}^{N times M}$ a real matrix with positive entries.



Can I find a condition to make $ operatorname{trace}(H) ge operatorname{trace}(B^{top}HB)$?










share|cite|improve this question











$endgroup$




Considering $H in mathbb{R}^{N times N}$ a real symmetric matrix which has both positive and negative eigenvalues, and $B in mathbb{R}^{N times M}$ a real matrix with positive entries.



Can I find a condition to make $ operatorname{trace}(H) ge operatorname{trace}(B^{top}HB)$?







linear-algebra matrices trace






share|cite|improve this question















share|cite|improve this question













share|cite|improve this question




share|cite|improve this question








edited Jan 20 at 19:17









Bernard

121k740116




121k740116










asked Jan 20 at 19:09









BabakBabak

342111




342111








  • 1




    $begingroup$
    Hint : You can suppose that H is diagonal. How ?
    $endgroup$
    – DLeMeur
    Jan 20 at 19:11










  • $begingroup$
    @DLeMeur. Well, I fiddled around with your hint, but couldn't get the answer! :D
    $endgroup$
    – Babak
    Jan 20 at 20:53










  • $begingroup$
    Rehint: How is the trace of $H $ related to its diagonalization?
    $endgroup$
    – Javi
    Jan 21 at 13:20












  • $begingroup$
    @Javi: Yes, i know the trace of $H$ is the sum of the diagonal entries of the diagonalized matrix, but i cannot link it to the rhs. Does it mean that $B^top HB$ has the same eigenvalues as of $H$ for any given tall matrix $B$?
    $endgroup$
    – Babak
    Jan 22 at 11:07










  • $begingroup$
    No, but you could use that to obtain some sufficient conditions for your inequality to hold
    $endgroup$
    – Javi
    Jan 23 at 2:55














  • 1




    $begingroup$
    Hint : You can suppose that H is diagonal. How ?
    $endgroup$
    – DLeMeur
    Jan 20 at 19:11










  • $begingroup$
    @DLeMeur. Well, I fiddled around with your hint, but couldn't get the answer! :D
    $endgroup$
    – Babak
    Jan 20 at 20:53










  • $begingroup$
    Rehint: How is the trace of $H $ related to its diagonalization?
    $endgroup$
    – Javi
    Jan 21 at 13:20












  • $begingroup$
    @Javi: Yes, i know the trace of $H$ is the sum of the diagonal entries of the diagonalized matrix, but i cannot link it to the rhs. Does it mean that $B^top HB$ has the same eigenvalues as of $H$ for any given tall matrix $B$?
    $endgroup$
    – Babak
    Jan 22 at 11:07










  • $begingroup$
    No, but you could use that to obtain some sufficient conditions for your inequality to hold
    $endgroup$
    – Javi
    Jan 23 at 2:55








1




1




$begingroup$
Hint : You can suppose that H is diagonal. How ?
$endgroup$
– DLeMeur
Jan 20 at 19:11




$begingroup$
Hint : You can suppose that H is diagonal. How ?
$endgroup$
– DLeMeur
Jan 20 at 19:11












$begingroup$
@DLeMeur. Well, I fiddled around with your hint, but couldn't get the answer! :D
$endgroup$
– Babak
Jan 20 at 20:53




$begingroup$
@DLeMeur. Well, I fiddled around with your hint, but couldn't get the answer! :D
$endgroup$
– Babak
Jan 20 at 20:53












$begingroup$
Rehint: How is the trace of $H $ related to its diagonalization?
$endgroup$
– Javi
Jan 21 at 13:20






$begingroup$
Rehint: How is the trace of $H $ related to its diagonalization?
$endgroup$
– Javi
Jan 21 at 13:20














$begingroup$
@Javi: Yes, i know the trace of $H$ is the sum of the diagonal entries of the diagonalized matrix, but i cannot link it to the rhs. Does it mean that $B^top HB$ has the same eigenvalues as of $H$ for any given tall matrix $B$?
$endgroup$
– Babak
Jan 22 at 11:07




$begingroup$
@Javi: Yes, i know the trace of $H$ is the sum of the diagonal entries of the diagonalized matrix, but i cannot link it to the rhs. Does it mean that $B^top HB$ has the same eigenvalues as of $H$ for any given tall matrix $B$?
$endgroup$
– Babak
Jan 22 at 11:07












$begingroup$
No, but you could use that to obtain some sufficient conditions for your inequality to hold
$endgroup$
– Javi
Jan 23 at 2:55




$begingroup$
No, but you could use that to obtain some sufficient conditions for your inequality to hold
$endgroup$
– Javi
Jan 23 at 2:55










4 Answers
4






active

oldest

votes


















0












$begingroup$

Since $H $ is real and symmetric, it can be diagonalized as $H = V^T D V $ where $V^T = V^{-1} $ and $ D$ is diagonal. Thus your expression may be rewritten as



$ tr (D) geq tr ( B^T H B ) = tr ( B B^T H ) $ where I have used that $tr (D) = tr (H) $ and the fact that the trace is invariant under cyclic permutations.



With this in mind, one obvious sufficient condition may be $B^{-1} = B^T $ in which case the equality holds.



Another example: pick $B $ such that $B B^T = H $ so $tr ( B B^T H) = tr ( H^2) = tr (D^2) $. So now you should pick $D $ such that $tr (D) geq tr (D^2)$, for example, having positive numbers smaller than 1 in the diagonal.



I'm aware these are not general conditions, only particular cases... but I guess this way you can build more examples and maybe get something more general.






share|cite|improve this answer









$endgroup$





















    0












    $begingroup$

    If you had looked at the case $n=1$ (and that's what you should have done), you would have seen that



    if $H>0$, then $1geq B^2$, if $H<0$, then $1leq B^2$ and if $H=0$, then every $B$ works.



    We assume that $tr(H)>0$.



    The condition is $tr(H))geq tr(HS)$ where $S=BB^T$ is $Ntimes N$ symmetric $geq 0$ with $>0$ entries. The required inequality stands if $S$ (or $B$) is small enough.



    More precisely, a sufficient condition is $tr(H^2)tr(S^2)leq (tr(H))^2$.



    Proof. $|tr(HS)|leq sqrt{tr(H^2)}sqrt{tr(S^2)}$ (by Cauchy-Schwartz).






    share|cite|improve this answer









    $endgroup$





















      0












      $begingroup$

      Step 1. Look at the eigendecomposition
      $$H=VDV^{rm T}$$



      $$
      H - text{symmetric} Rightarrow begin{cases} D - text{diagonal} \ V - text{orthogonal} \end{cases}
      $$



      Step 2. Come up with any value $Delta$ you want $-$ that will be your future $operatorname{trace}(B^{rm T}HB)$.



      Step 3. Choose such $c_kge0$ that $sum c_klambda_k= Delta$, where $lambda_k$ is the diagonal entries of $D$. It is always possible since the matrix $H$ has both positive and negative eigenvalues.



      Step 4a. Take a diagonal matrix $C$ with diagonal entries $sqrt{c_k}$ .



      Step 4b. If you want matrix $B$ to have exactly $M$ columns ($M<N$) then you have to pick exactly $M$ nonzero elements $c_k$ on Step 3 and then drop $N-M$ zero columns of matrix $C$ obtained on Step 4a, leaving yourself with the new matrix $C$ of size $N times M$.



      Step 5. Take $B=VC$ and get $operatorname{trace}(B^{rm T}HB)=Delta$.



      The last is true because
      $$
      B^{rm T}HB=B^{rm T}VDV^{rm T}B=C^{rm T} color{purple}{ V^{rm T}V}Dcolor{purple}{V^{rm T}V}C=C^{rm T}DC,
      $$



      which is a diagonal matrix with ${c_klambda_k}$ being its diagonal entries.



      Unfortunately, this is not a proper answer to your question since I drop the restriction of $B$ being entrywise positive. I realized it after posting the answer but decided to leave it as it is.






      share|cite|improve this answer











      $endgroup$





















        0












        $begingroup$

        Let $H = UDU^T$ where $U$ contains the eigenvectors and $D$ is diagonal containing eigenvalues. Let's choose $B = UW^T$, then
        $$B^T H B = (UW^T)^T(UDU^T) (UW^T) = W U^T UDU^TUW^T = WDW^T $$
        Choosing $W$ as diagonal, we get that $$operatorname{trace}(H) ge operatorname{trace}(B^{top}HB) iff sum d_k geq sum w_k^2 d_k$$
        Since $d_k$'s could be negative and positive, then choose $w_k = 1$ if $d_k <0$, else $w_k = 0$. This means that $operatorname{trace}(B^{top}HB)$ is just summing the negative eigenvalues of $H$ , if any.






        share|cite|improve this answer











        $endgroup$













          Your Answer





          StackExchange.ifUsing("editor", function () {
          return StackExchange.using("mathjaxEditing", function () {
          StackExchange.MarkdownEditor.creationCallbacks.add(function (editor, postfix) {
          StackExchange.mathjaxEditing.prepareWmdForMathJax(editor, postfix, [["$", "$"], ["\\(","\\)"]]);
          });
          });
          }, "mathjax-editing");

          StackExchange.ready(function() {
          var channelOptions = {
          tags: "".split(" "),
          id: "69"
          };
          initTagRenderer("".split(" "), "".split(" "), channelOptions);

          StackExchange.using("externalEditor", function() {
          // Have to fire editor after snippets, if snippets enabled
          if (StackExchange.settings.snippets.snippetsEnabled) {
          StackExchange.using("snippets", function() {
          createEditor();
          });
          }
          else {
          createEditor();
          }
          });

          function createEditor() {
          StackExchange.prepareEditor({
          heartbeatType: 'answer',
          autoActivateHeartbeat: false,
          convertImagesToLinks: true,
          noModals: true,
          showLowRepImageUploadWarning: true,
          reputationToPostImages: 10,
          bindNavPrevention: true,
          postfix: "",
          imageUploader: {
          brandingHtml: "Powered by u003ca class="icon-imgur-white" href="https://imgur.com/"u003eu003c/au003e",
          contentPolicyHtml: "User contributions licensed under u003ca href="https://creativecommons.org/licenses/by-sa/3.0/"u003ecc by-sa 3.0 with attribution requiredu003c/au003e u003ca href="https://stackoverflow.com/legal/content-policy"u003e(content policy)u003c/au003e",
          allowUrls: true
          },
          noCode: true, onDemand: true,
          discardSelector: ".discard-answer"
          ,immediatelyShowMarkdownHelp:true
          });


          }
          });














          draft saved

          draft discarded


















          StackExchange.ready(
          function () {
          StackExchange.openid.initPostLogin('.new-post-login', 'https%3a%2f%2fmath.stackexchange.com%2fquestions%2f3081016%2fhow-to-choose-b-to-have-operatornametraceh-ge-operatornametraceb%23new-answer', 'question_page');
          }
          );

          Post as a guest















          Required, but never shown

























          4 Answers
          4






          active

          oldest

          votes








          4 Answers
          4






          active

          oldest

          votes









          active

          oldest

          votes






          active

          oldest

          votes









          0












          $begingroup$

          Since $H $ is real and symmetric, it can be diagonalized as $H = V^T D V $ where $V^T = V^{-1} $ and $ D$ is diagonal. Thus your expression may be rewritten as



          $ tr (D) geq tr ( B^T H B ) = tr ( B B^T H ) $ where I have used that $tr (D) = tr (H) $ and the fact that the trace is invariant under cyclic permutations.



          With this in mind, one obvious sufficient condition may be $B^{-1} = B^T $ in which case the equality holds.



          Another example: pick $B $ such that $B B^T = H $ so $tr ( B B^T H) = tr ( H^2) = tr (D^2) $. So now you should pick $D $ such that $tr (D) geq tr (D^2)$, for example, having positive numbers smaller than 1 in the diagonal.



          I'm aware these are not general conditions, only particular cases... but I guess this way you can build more examples and maybe get something more general.






          share|cite|improve this answer









          $endgroup$


















            0












            $begingroup$

            Since $H $ is real and symmetric, it can be diagonalized as $H = V^T D V $ where $V^T = V^{-1} $ and $ D$ is diagonal. Thus your expression may be rewritten as



            $ tr (D) geq tr ( B^T H B ) = tr ( B B^T H ) $ where I have used that $tr (D) = tr (H) $ and the fact that the trace is invariant under cyclic permutations.



            With this in mind, one obvious sufficient condition may be $B^{-1} = B^T $ in which case the equality holds.



            Another example: pick $B $ such that $B B^T = H $ so $tr ( B B^T H) = tr ( H^2) = tr (D^2) $. So now you should pick $D $ such that $tr (D) geq tr (D^2)$, for example, having positive numbers smaller than 1 in the diagonal.



            I'm aware these are not general conditions, only particular cases... but I guess this way you can build more examples and maybe get something more general.






            share|cite|improve this answer









            $endgroup$
















              0












              0








              0





              $begingroup$

              Since $H $ is real and symmetric, it can be diagonalized as $H = V^T D V $ where $V^T = V^{-1} $ and $ D$ is diagonal. Thus your expression may be rewritten as



              $ tr (D) geq tr ( B^T H B ) = tr ( B B^T H ) $ where I have used that $tr (D) = tr (H) $ and the fact that the trace is invariant under cyclic permutations.



              With this in mind, one obvious sufficient condition may be $B^{-1} = B^T $ in which case the equality holds.



              Another example: pick $B $ such that $B B^T = H $ so $tr ( B B^T H) = tr ( H^2) = tr (D^2) $. So now you should pick $D $ such that $tr (D) geq tr (D^2)$, for example, having positive numbers smaller than 1 in the diagonal.



              I'm aware these are not general conditions, only particular cases... but I guess this way you can build more examples and maybe get something more general.






              share|cite|improve this answer









              $endgroup$



              Since $H $ is real and symmetric, it can be diagonalized as $H = V^T D V $ where $V^T = V^{-1} $ and $ D$ is diagonal. Thus your expression may be rewritten as



              $ tr (D) geq tr ( B^T H B ) = tr ( B B^T H ) $ where I have used that $tr (D) = tr (H) $ and the fact that the trace is invariant under cyclic permutations.



              With this in mind, one obvious sufficient condition may be $B^{-1} = B^T $ in which case the equality holds.



              Another example: pick $B $ such that $B B^T = H $ so $tr ( B B^T H) = tr ( H^2) = tr (D^2) $. So now you should pick $D $ such that $tr (D) geq tr (D^2)$, for example, having positive numbers smaller than 1 in the diagonal.



              I'm aware these are not general conditions, only particular cases... but I guess this way you can build more examples and maybe get something more general.







              share|cite|improve this answer












              share|cite|improve this answer



              share|cite|improve this answer










              answered Jan 23 at 3:15









              JaviJavi

              3979




              3979























                  0












                  $begingroup$

                  If you had looked at the case $n=1$ (and that's what you should have done), you would have seen that



                  if $H>0$, then $1geq B^2$, if $H<0$, then $1leq B^2$ and if $H=0$, then every $B$ works.



                  We assume that $tr(H)>0$.



                  The condition is $tr(H))geq tr(HS)$ where $S=BB^T$ is $Ntimes N$ symmetric $geq 0$ with $>0$ entries. The required inequality stands if $S$ (or $B$) is small enough.



                  More precisely, a sufficient condition is $tr(H^2)tr(S^2)leq (tr(H))^2$.



                  Proof. $|tr(HS)|leq sqrt{tr(H^2)}sqrt{tr(S^2)}$ (by Cauchy-Schwartz).






                  share|cite|improve this answer









                  $endgroup$


















                    0












                    $begingroup$

                    If you had looked at the case $n=1$ (and that's what you should have done), you would have seen that



                    if $H>0$, then $1geq B^2$, if $H<0$, then $1leq B^2$ and if $H=0$, then every $B$ works.



                    We assume that $tr(H)>0$.



                    The condition is $tr(H))geq tr(HS)$ where $S=BB^T$ is $Ntimes N$ symmetric $geq 0$ with $>0$ entries. The required inequality stands if $S$ (or $B$) is small enough.



                    More precisely, a sufficient condition is $tr(H^2)tr(S^2)leq (tr(H))^2$.



                    Proof. $|tr(HS)|leq sqrt{tr(H^2)}sqrt{tr(S^2)}$ (by Cauchy-Schwartz).






                    share|cite|improve this answer









                    $endgroup$
















                      0












                      0








                      0





                      $begingroup$

                      If you had looked at the case $n=1$ (and that's what you should have done), you would have seen that



                      if $H>0$, then $1geq B^2$, if $H<0$, then $1leq B^2$ and if $H=0$, then every $B$ works.



                      We assume that $tr(H)>0$.



                      The condition is $tr(H))geq tr(HS)$ where $S=BB^T$ is $Ntimes N$ symmetric $geq 0$ with $>0$ entries. The required inequality stands if $S$ (or $B$) is small enough.



                      More precisely, a sufficient condition is $tr(H^2)tr(S^2)leq (tr(H))^2$.



                      Proof. $|tr(HS)|leq sqrt{tr(H^2)}sqrt{tr(S^2)}$ (by Cauchy-Schwartz).






                      share|cite|improve this answer









                      $endgroup$



                      If you had looked at the case $n=1$ (and that's what you should have done), you would have seen that



                      if $H>0$, then $1geq B^2$, if $H<0$, then $1leq B^2$ and if $H=0$, then every $B$ works.



                      We assume that $tr(H)>0$.



                      The condition is $tr(H))geq tr(HS)$ where $S=BB^T$ is $Ntimes N$ symmetric $geq 0$ with $>0$ entries. The required inequality stands if $S$ (or $B$) is small enough.



                      More precisely, a sufficient condition is $tr(H^2)tr(S^2)leq (tr(H))^2$.



                      Proof. $|tr(HS)|leq sqrt{tr(H^2)}sqrt{tr(S^2)}$ (by Cauchy-Schwartz).







                      share|cite|improve this answer












                      share|cite|improve this answer



                      share|cite|improve this answer










                      answered Jan 23 at 11:07









                      loup blancloup blanc

                      23.4k21851




                      23.4k21851























                          0












                          $begingroup$

                          Step 1. Look at the eigendecomposition
                          $$H=VDV^{rm T}$$



                          $$
                          H - text{symmetric} Rightarrow begin{cases} D - text{diagonal} \ V - text{orthogonal} \end{cases}
                          $$



                          Step 2. Come up with any value $Delta$ you want $-$ that will be your future $operatorname{trace}(B^{rm T}HB)$.



                          Step 3. Choose such $c_kge0$ that $sum c_klambda_k= Delta$, where $lambda_k$ is the diagonal entries of $D$. It is always possible since the matrix $H$ has both positive and negative eigenvalues.



                          Step 4a. Take a diagonal matrix $C$ with diagonal entries $sqrt{c_k}$ .



                          Step 4b. If you want matrix $B$ to have exactly $M$ columns ($M<N$) then you have to pick exactly $M$ nonzero elements $c_k$ on Step 3 and then drop $N-M$ zero columns of matrix $C$ obtained on Step 4a, leaving yourself with the new matrix $C$ of size $N times M$.



                          Step 5. Take $B=VC$ and get $operatorname{trace}(B^{rm T}HB)=Delta$.



                          The last is true because
                          $$
                          B^{rm T}HB=B^{rm T}VDV^{rm T}B=C^{rm T} color{purple}{ V^{rm T}V}Dcolor{purple}{V^{rm T}V}C=C^{rm T}DC,
                          $$



                          which is a diagonal matrix with ${c_klambda_k}$ being its diagonal entries.



                          Unfortunately, this is not a proper answer to your question since I drop the restriction of $B$ being entrywise positive. I realized it after posting the answer but decided to leave it as it is.






                          share|cite|improve this answer











                          $endgroup$


















                            0












                            $begingroup$

                            Step 1. Look at the eigendecomposition
                            $$H=VDV^{rm T}$$



                            $$
                            H - text{symmetric} Rightarrow begin{cases} D - text{diagonal} \ V - text{orthogonal} \end{cases}
                            $$



                            Step 2. Come up with any value $Delta$ you want $-$ that will be your future $operatorname{trace}(B^{rm T}HB)$.



                            Step 3. Choose such $c_kge0$ that $sum c_klambda_k= Delta$, where $lambda_k$ is the diagonal entries of $D$. It is always possible since the matrix $H$ has both positive and negative eigenvalues.



                            Step 4a. Take a diagonal matrix $C$ with diagonal entries $sqrt{c_k}$ .



                            Step 4b. If you want matrix $B$ to have exactly $M$ columns ($M<N$) then you have to pick exactly $M$ nonzero elements $c_k$ on Step 3 and then drop $N-M$ zero columns of matrix $C$ obtained on Step 4a, leaving yourself with the new matrix $C$ of size $N times M$.



                            Step 5. Take $B=VC$ and get $operatorname{trace}(B^{rm T}HB)=Delta$.



                            The last is true because
                            $$
                            B^{rm T}HB=B^{rm T}VDV^{rm T}B=C^{rm T} color{purple}{ V^{rm T}V}Dcolor{purple}{V^{rm T}V}C=C^{rm T}DC,
                            $$



                            which is a diagonal matrix with ${c_klambda_k}$ being its diagonal entries.



                            Unfortunately, this is not a proper answer to your question since I drop the restriction of $B$ being entrywise positive. I realized it after posting the answer but decided to leave it as it is.






                            share|cite|improve this answer











                            $endgroup$
















                              0












                              0








                              0





                              $begingroup$

                              Step 1. Look at the eigendecomposition
                              $$H=VDV^{rm T}$$



                              $$
                              H - text{symmetric} Rightarrow begin{cases} D - text{diagonal} \ V - text{orthogonal} \end{cases}
                              $$



                              Step 2. Come up with any value $Delta$ you want $-$ that will be your future $operatorname{trace}(B^{rm T}HB)$.



                              Step 3. Choose such $c_kge0$ that $sum c_klambda_k= Delta$, where $lambda_k$ is the diagonal entries of $D$. It is always possible since the matrix $H$ has both positive and negative eigenvalues.



                              Step 4a. Take a diagonal matrix $C$ with diagonal entries $sqrt{c_k}$ .



                              Step 4b. If you want matrix $B$ to have exactly $M$ columns ($M<N$) then you have to pick exactly $M$ nonzero elements $c_k$ on Step 3 and then drop $N-M$ zero columns of matrix $C$ obtained on Step 4a, leaving yourself with the new matrix $C$ of size $N times M$.



                              Step 5. Take $B=VC$ and get $operatorname{trace}(B^{rm T}HB)=Delta$.



                              The last is true because
                              $$
                              B^{rm T}HB=B^{rm T}VDV^{rm T}B=C^{rm T} color{purple}{ V^{rm T}V}Dcolor{purple}{V^{rm T}V}C=C^{rm T}DC,
                              $$



                              which is a diagonal matrix with ${c_klambda_k}$ being its diagonal entries.



                              Unfortunately, this is not a proper answer to your question since I drop the restriction of $B$ being entrywise positive. I realized it after posting the answer but decided to leave it as it is.






                              share|cite|improve this answer











                              $endgroup$



                              Step 1. Look at the eigendecomposition
                              $$H=VDV^{rm T}$$



                              $$
                              H - text{symmetric} Rightarrow begin{cases} D - text{diagonal} \ V - text{orthogonal} \end{cases}
                              $$



                              Step 2. Come up with any value $Delta$ you want $-$ that will be your future $operatorname{trace}(B^{rm T}HB)$.



                              Step 3. Choose such $c_kge0$ that $sum c_klambda_k= Delta$, where $lambda_k$ is the diagonal entries of $D$. It is always possible since the matrix $H$ has both positive and negative eigenvalues.



                              Step 4a. Take a diagonal matrix $C$ with diagonal entries $sqrt{c_k}$ .



                              Step 4b. If you want matrix $B$ to have exactly $M$ columns ($M<N$) then you have to pick exactly $M$ nonzero elements $c_k$ on Step 3 and then drop $N-M$ zero columns of matrix $C$ obtained on Step 4a, leaving yourself with the new matrix $C$ of size $N times M$.



                              Step 5. Take $B=VC$ and get $operatorname{trace}(B^{rm T}HB)=Delta$.



                              The last is true because
                              $$
                              B^{rm T}HB=B^{rm T}VDV^{rm T}B=C^{rm T} color{purple}{ V^{rm T}V}Dcolor{purple}{V^{rm T}V}C=C^{rm T}DC,
                              $$



                              which is a diagonal matrix with ${c_klambda_k}$ being its diagonal entries.



                              Unfortunately, this is not a proper answer to your question since I drop the restriction of $B$ being entrywise positive. I realized it after posting the answer but decided to leave it as it is.







                              share|cite|improve this answer














                              share|cite|improve this answer



                              share|cite|improve this answer








                              edited Jan 23 at 13:21

























                              answered Jan 23 at 12:48









                              ZeeklessZeekless

                              577111




                              577111























                                  0












                                  $begingroup$

                                  Let $H = UDU^T$ where $U$ contains the eigenvectors and $D$ is diagonal containing eigenvalues. Let's choose $B = UW^T$, then
                                  $$B^T H B = (UW^T)^T(UDU^T) (UW^T) = W U^T UDU^TUW^T = WDW^T $$
                                  Choosing $W$ as diagonal, we get that $$operatorname{trace}(H) ge operatorname{trace}(B^{top}HB) iff sum d_k geq sum w_k^2 d_k$$
                                  Since $d_k$'s could be negative and positive, then choose $w_k = 1$ if $d_k <0$, else $w_k = 0$. This means that $operatorname{trace}(B^{top}HB)$ is just summing the negative eigenvalues of $H$ , if any.






                                  share|cite|improve this answer











                                  $endgroup$


















                                    0












                                    $begingroup$

                                    Let $H = UDU^T$ where $U$ contains the eigenvectors and $D$ is diagonal containing eigenvalues. Let's choose $B = UW^T$, then
                                    $$B^T H B = (UW^T)^T(UDU^T) (UW^T) = W U^T UDU^TUW^T = WDW^T $$
                                    Choosing $W$ as diagonal, we get that $$operatorname{trace}(H) ge operatorname{trace}(B^{top}HB) iff sum d_k geq sum w_k^2 d_k$$
                                    Since $d_k$'s could be negative and positive, then choose $w_k = 1$ if $d_k <0$, else $w_k = 0$. This means that $operatorname{trace}(B^{top}HB)$ is just summing the negative eigenvalues of $H$ , if any.






                                    share|cite|improve this answer











                                    $endgroup$
















                                      0












                                      0








                                      0





                                      $begingroup$

                                      Let $H = UDU^T$ where $U$ contains the eigenvectors and $D$ is diagonal containing eigenvalues. Let's choose $B = UW^T$, then
                                      $$B^T H B = (UW^T)^T(UDU^T) (UW^T) = W U^T UDU^TUW^T = WDW^T $$
                                      Choosing $W$ as diagonal, we get that $$operatorname{trace}(H) ge operatorname{trace}(B^{top}HB) iff sum d_k geq sum w_k^2 d_k$$
                                      Since $d_k$'s could be negative and positive, then choose $w_k = 1$ if $d_k <0$, else $w_k = 0$. This means that $operatorname{trace}(B^{top}HB)$ is just summing the negative eigenvalues of $H$ , if any.






                                      share|cite|improve this answer











                                      $endgroup$



                                      Let $H = UDU^T$ where $U$ contains the eigenvectors and $D$ is diagonal containing eigenvalues. Let's choose $B = UW^T$, then
                                      $$B^T H B = (UW^T)^T(UDU^T) (UW^T) = W U^T UDU^TUW^T = WDW^T $$
                                      Choosing $W$ as diagonal, we get that $$operatorname{trace}(H) ge operatorname{trace}(B^{top}HB) iff sum d_k geq sum w_k^2 d_k$$
                                      Since $d_k$'s could be negative and positive, then choose $w_k = 1$ if $d_k <0$, else $w_k = 0$. This means that $operatorname{trace}(B^{top}HB)$ is just summing the negative eigenvalues of $H$ , if any.







                                      share|cite|improve this answer














                                      share|cite|improve this answer



                                      share|cite|improve this answer








                                      edited Jan 23 at 13:48

























                                      answered Jan 23 at 13:38









                                      Ahmad BazziAhmad Bazzi

                                      8,3282824




                                      8,3282824






























                                          draft saved

                                          draft discarded




















































                                          Thanks for contributing an answer to Mathematics Stack Exchange!


                                          • Please be sure to answer the question. Provide details and share your research!

                                          But avoid



                                          • Asking for help, clarification, or responding to other answers.

                                          • Making statements based on opinion; back them up with references or personal experience.


                                          Use MathJax to format equations. MathJax reference.


                                          To learn more, see our tips on writing great answers.




                                          draft saved


                                          draft discarded














                                          StackExchange.ready(
                                          function () {
                                          StackExchange.openid.initPostLogin('.new-post-login', 'https%3a%2f%2fmath.stackexchange.com%2fquestions%2f3081016%2fhow-to-choose-b-to-have-operatornametraceh-ge-operatornametraceb%23new-answer', 'question_page');
                                          }
                                          );

                                          Post as a guest















                                          Required, but never shown





















































                                          Required, but never shown














                                          Required, but never shown












                                          Required, but never shown







                                          Required, but never shown

































                                          Required, but never shown














                                          Required, but never shown












                                          Required, but never shown







                                          Required, but never shown







                                          Popular posts from this blog

                                          Mario Kart Wii

                                          What does “Dominus providebit” mean?

                                          Antonio Litta Visconti Arese